Need some assistance with a few calculus questions

Click For Summary
The discussion revolves around a user seeking help with calculus problems, specifically evaluating integrals and determining when a particle reaches a certain velocity. The user provided answers but requested step-by-step solutions due to confusion with their own calculations. Suggestions included splitting integrals into simpler parts and using substitutions for easier evaluation. The user clarified their approach to finding the time when the particle reaches a velocity of 52 m/s, ultimately correcting their algebra. The thread emphasizes the importance of showing detailed working for clarity in solving calculus problems.
russjai
Messages
7
Reaction score
0
Hi everyone, I have the answers for the following questions, I need the working out shown step by step. As i seem to get the incorrect answer. Not sure how to even start with the evaluation of integrals when in fraction form as below. Help very appreciated . Thanks

1. Evaluate the integrals

a.) (x^2+6)/(√x) dx (between 1 and 4)
ans = 24.4

b.) (2x)/(√1+2x) dx (between 0 and 4)
ans = 20/3


2.) The position function (in meters) of a particle is given by s=t^3-10.5t^2-2t t>0 . When does the particle reach a velocity of 52 m/s?

ans = 9



3. The Attempt at a Solution for Q.2

started by taking the derivative of s=t^3-10.5t^2-2t, Which gives Sv=3t^2-21t-2 .

I seem to get the wrong answer using algebra to solve t for 52= 3t^2-21t-2 .

Thanks in advance for your help.

Cheers
 
Physics news on Phys.org
welcome to pf!

hi russjai! welcome to pf! :smile:

(try using the X2 icon just above the Reply box :wink:)

for 1a., split it into x2 /√x and 1/√x :wink:

for 1b., do a substitution first, to make it like (a)
russjai said:
I seem to get the wrong answer using algebra to solve t for 52= 3t^2-21t-2 .

the method looks looks ok …

that's t2 - 7t - 18 = 0 …

what did you get?​
 
Thanks for your quick response tiny-tim,

Ok i see where i went wrong with the algebra concerning Q.2 . I can now solve it to
t(t-7)=18 , Where 't' obviously = 9

For the 1st part regarding evaluating the integrals, still not too sure how to go about it would you possibly be able to show the working for Q.1) b.) (2x)/(√1+2x) dx (between 0 and 4)
From the start all the way to the answer 20/3 . ?

That would be extremely helpful.

Cheers
 
russjai said:
… would you possibly be able to show the working for Q.1) b.)

not on this forum :redface:

make a substituion …

show us what you get :smile:
 
Question: A clock's minute hand has length 4 and its hour hand has length 3. What is the distance between the tips at the moment when it is increasing most rapidly?(Putnam Exam Question) Answer: Making assumption that both the hands moves at constant angular velocities, the answer is ## \sqrt{7} .## But don't you think this assumption is somewhat doubtful and wrong?

Similar threads

  • · Replies 2 ·
Replies
2
Views
1K
  • · Replies 15 ·
Replies
15
Views
2K
  • · Replies 10 ·
Replies
10
Views
2K
  • · Replies 2 ·
Replies
2
Views
2K
Replies
12
Views
2K
  • · Replies 6 ·
Replies
6
Views
2K
  • · Replies 13 ·
Replies
13
Views
2K
  • · Replies 25 ·
Replies
25
Views
2K
  • · Replies 13 ·
Replies
13
Views
1K
  • · Replies 1 ·
Replies
1
Views
2K